Đăng ký Đăng nhập
Trang chủ Phương trình hàm và một số tính chất cực trị của hàm số học...

Tài liệu Phương trình hàm và một số tính chất cực trị của hàm số học

.PDF
41
4
99

Mô tả:

.. ĐẠI HỌC THÁI NGUYÊN TRƯỜNG ĐẠI HỌC KHOA HỌC LẠI THỊ THÚY HẢI PHƯƠNG TRÌNH HÀM VÀ MỘT SỐ TÍNH CHẤT CỰC TRỊ CỦA HÀM SỐ HỌC LUẬN VĂN THẠC SĨ TOÁN HỌC Thái Nguyên - 2018 ĐẠI HỌC THÁI NGUYÊN TRƯỜNG ĐẠI HỌC KHOA HỌC LẠI THỊ THÚY HẢI PHƯƠNG TRÌNH HÀM VÀ MỘT SỐ TÍNH CHẤT CỰC TRỊ CỦA HÀM SỐ HỌC LUẬN VĂN THẠC SĨ TOÁN HỌC Chuyên ngành: Phương pháp Toán sơ cấp Mã số: 8460113 NGƯỜI HƯỚNG DẪN KHOA HỌC GS.TSKH. HÀ HUY KHOÁI Thái Nguyên - 2018 3 Mục lục Danh sách kí hiệu 5 Mở đầu 6 1 Phương trình hàm đối với hàm tổng các ước 8 1.1 Giới thiệu . . . . . . . . . . . . . . . . . . . . . . . . . . . . . . . . . 1.2 Một số ký hiệu và kiến thức chuẩn bị . . . . . . . . . . . . . . . . . . 10 1.3 Cấu trúc của nghiệm . . . . . . . . . . . . . . . . . . . . . . . . . . . 12 1.4 Nghiệm với ω(n) 6 4 . . . . . . . . . . . . . . . . . . . . . . . . . . . 13 1.5 Trường hợp n không có ước là luỹ thừa bậc 4 . . . . . . . . . . . . . 17 1.6 Đếm các phần tử trong K ∩ [1, x] . . . . . . . . . . . . . . . . . . . . 20 1.7 Kết luận Chương 1 . . . . . . . . . . . . . . . . . . . . . . . . . . . . 24 2 Bậc cực trị của một số hàm số học 8 26 2.1 Giới thiệu . . . . . . . . . . . . . . . . . . . . . . . . . . . . . . . . . 26 2.2 Chuỗi Dirichlet của Vk (n) . . . . . . . . . . . . . . . . . . . . . . . . 28 2.3 Bậc cực trị liên quan đến các hàm số học suy rộng cổ điển . . . . . . 30 2.4 Bậc cực trị liên quan đến các tương tự đơn của σk và φk . . . . . . . 31 2.5 Bậc cực trị liên quan đến hợp các hàm số học . . . . . . . . . . . . . 33 2.6 Các bài toán mở . . . . . . . . . . . . . . . . . . . . . . . . . . . . . 38 2.7 Kết luận Chương 2 . . . . . . . . . . . . . . . . . . . . . . . . . . . . 39 Kết luận 40 Tài liệu tham khảo 41 4 Lời cảm ơn Trước hết, tác giả muốn tỏ lòng biết ơn đến người hướng dẫn khoa học của mình, GS.TSKH. Hà Huy Khoái (Trường Đại học Thăng Long), người đã đặt bài toán của đề tài, tận tình hướng dẫn để luận văn này được hoàn thành tốt đẹp. Nhân dịp này, tác giả xin được cảm ơn Ban Giám hiệu Trường Đại học Khoa học - Đại học Thái Nguyên, Ban Chủ nhiệm Khoa Toán–Tin, cùng các giảng viên đã tham gia giảng dạy lớp Cao học Toán khóa 10 (2016-2018). Xin trân trọng cảm ơn Sở Giáo dục và Đào tạo Hải Phòng, Ban Giám hiệu và các đồng nghiệp ở Trường THPT Phạm Ngũ Lão, Thủy Nguyên, Hải Phòng, đã tạo mọi điều kiện thuận lợi để tác giả học tập và nghiên cứu. Lời cuối cùng, tác giả muốn dành để tri ân bố mẹ và gia đình vì đã chia sẻ những khó khăn để tác giả hoàn thành công việc học tập của mình. 5 Danh sách kí hiệu #X lực lượng của tập hợp X dxe trần của số x bxc sàn của số x a|b b là bội của a a6| b a không phải là ước của b σ(n) tổng các ước của n vp (n) lũy thừa cao nhất của p chia hết n  φ(n) hàm Euler, φ(n) = n Q p|n 1− 1 p   ζ(s) hàm zeta (ζ) Riemann, ζ(s) = Q p s = σ + it ∈ C và σ > 1 lim sup giới hạn trên lim inf giới hạn dưới 1− 1 ps −1 , 6 Mở đầu Có thể nói, Lý thuyết số là một ngành khoa học sớm nhất của nhân loại. Trước những năm 70 của thế kỷ XX, Lý thuyết số được coi là một ngành thuần túy lý thuyết, còn hiện nay Lý thuyết số đang trở thành một trong những lĩnh vực có nhiều ứng dụng sôi động nhất của Toán học. Trong Lý thuyết số, các hàm số học là những hàm số xác định trên tập hợp các số tự nhiên và có tập giá trị là một tập con nào đó của tập hợp các số phức. Các điều kiện được đặt lên các hàm số học sẽ phụ thuộc vào mục đích nghiên cứu. Như Hardy & Wright từng yêu cầu, một hàm số học phải “thể hiện một số tính số học của n”. Luận văn này có mục đích nghiên cứu một mối quan hệ về hàm số học là tổng các ước của một số nguyên cho trước, và sau cùng là bậc cực trị của một số lớp hàm số học quan trọng. Ngoài các phần Mở đầu, Kết luận, Tài liệu tham khảo, nội dung của luận văn được trình bày trong hai chương: • Chương 1. Phương trình hàm đối với hàm tổng các ước. Nội dung của chương này là nghiên cứu về nghiệm nguyên dương của σ(n) = γ(n)2 , trong đó σ(n) và γ(n) tương ứng là tổng của các ước và tích của các ước nguyên tố phân biệt của n. • Chương 2. Bậc cực trị của một số hàm số học. Chương này dành để trình bày về để chuỗi Dirichlet của V (n) (số các số chính quy modulo n) và xác định các bậc cực trị của một số hàm số học cổ điển, các hàm tổng 7 các ước đơn của n (ước d của n được gọi là đơn nếu n và n/d nguyên tố cùng nhau) và liên hệ với hàm φ-Euler. Thái Nguyên, ngày 22 tháng 4 năm 2018 Tác giả Lại Thị Thúy Hải 8 Chương 1 Phương trình hàm đối với hàm tổng các ước Chương này dành để nghiên cứu các số nguyên n > 1 thỏa mãn quan hệ σ(n) = γ(n)2 , trong đó σ(n) và γ(n) tương ứng là tổng các ước và tích các ước nguyên tố phân biệt của n. Ta sẽ chứng minh rằng nghiệm có không quá bốn ước nguyên tố phân biệt duy nhất là n = 1782. Ta cũng sẽ chỉ ra không tồn tại nghiệm nào không có ước là lũy thừa bậc 4, và số nghiệm nhỏ hơn x là không vượt quá x1/4+ với  > 0 và với mọi x > x . Thêm nữa, số n được gọi là nguyên thủy nếu không có ước đơn thực sự d nào của n thỏa mãn σ(d) | γ(d)2 . Ta sẽ chỉ ra số nghiệm nguyên thủy của phương trình không vượt quá x là nhỏ hơn x với x > x . Nội dung chương này được viết dựa vào tài liệu Broughan A.K. et al [3]. 1.1 Giới thiệu Tại hội nghị khoa học “Western Number Theory Conference” 1 năm 2000, De Koninck J.-M. (tác giả thứ hai của công trình Broughan A.K. et al [3]) đưa ra câu hỏi về tìm nghiệm nguyên dương n của phương trình σ(n) = γ(n)2 1 Hội nghị Lý thuyết số Bờ Tây, https://westcoastnumbertheory.org/ (1.1) 9 (gọi là “phương trình De Koninck”), trong đó σ(n) là tổng tất cả các ước dương của n, và γ(n) là tích tất cả các ước nguyên tố phân biệt của n, cũng được gọi là “cốt lõi” (core) của n. Dễ thấy, n = 1 và n = 1782 là các nghiệm, nhưng, tình đến năm 2012 - năm xuất bản công trình Broughan A.K. et al [3] - người ta không biết thêm một nghiệm nào nữa. Một tìm kiếm bằng máy tính với mọi n 6 1011 không cho thấy nghiệm nào khác. Một giả thuyết tự nhiên (được gọi là “Giả thuyết De Koninck”) là phương trình này không có nghiệm nào khác. Nó được trình bày bởi Guy R.K. (2004). Có một kết quả đã được chứng minh rằng một nghiệm bất kỳ không tầm thường n phải có ít nhất ba ước nguyên tố, chẵn, và không thể là số hoàn toàn không chính phương (tạm dịch thuật ngữ squarefree - số không có ước chính phương khác 1). Luca F. (2004) chỉ ra rằng số nghiệm mà số các ước nguyên tố là số cố định cho trước chỉ là hữu hạn. Thật ra Luca F. đã chứng điều này cho lớp rộng hơn các nghiệm dương n của phương trình σ(n) = aγ(n)K trong đó K > 2 và 1 6 a 6 L với K và L là các tham số cố định. Tuy nhiên, có rất ít tiến bộ trong nghiên cứu Giả thuyết De Koninck. Ở đây, luận văn sẽ trình bày kết quả nói rằng các nghiệm n = 1, 1782 là các số duy nhất có ω(n) 6 4, trong đó như thường lệ, ω(n) là số ước nguyên tố phân biệt của n. Phương pháp chứng minh dựa vào các chặn trên sơ cấp đối với những số mũ có thể của các số nguyên tố xuất hiện trong phân tích của n, và sử dụng kết thức để giải hệ phương trình đa thức thu được, mà các ẩn là các ước nguyên tố của n. Ta chứng minh rằng nếu một số nguyên n là không có ước là luỹ thữa bậc 4 (fourth power free) (tức là p4 - n với mọi số nguyên tố p), thì n không thể thỏa mãn phương trình De Koninck (1.1). Sau đó chúng ta đếm số nghiệm tiềm năng n không vượt quá x. Pollack & Pomerance [4], gọi một số nguyên dương n là nguyên tố–hoàn hảo (prime–perfect) nếu n và σ(n) chung nhau tập hợp các ước nguyên tố. Rõ ràng, mọi nghiệm n của phương trình De 10 Koninck là số nguyên tố–hoàn hảo. Pollack & Pomerance chứng minh rằng tập hợp các số nguyên tố–hoàn hảo là vô hạn, và hàm đếm các số nguyên tố–hoàn hảo n 6 x có lực lượng nhiều nhất là x1/3+o(1) khi x → ∞. Sử dụng các kết quả của Pollack & Pomerance, ta sẽ chứng minh rằng số các nghiệm n 6 x của phương trình De Koninck nhiều nhất là x1/4+ với bất kỳ  > 0 và mọi x > x . Bằng cách hạn chế đến nghiệm “nguyên thủy” (“primitive” solutions) sử dụng phương pháp của Wirsing E., ta nhận được một chặn trên của O(x ) với mọi  > 0. Khái niệm “nguyên thủy” được sử dụng ở đây để chỉ các số không có ước đơn thực sự d | n thỏa mãn σ(d) | γ(d)2 . Cuối cùng ta có một số nhận xét về bài toán liên quan là xác định các số số nguyên n sao cho γ(n)2 | σ(n). Tóm lại, mục đích của chương này là trình bày một số sự kiện ủng hộ cho Giả thuyết De Konnick, và chỉ ra cấu trúc cần thiết cho một phản ví dụ, nếu có. Mọi nghiệm không tầm thường khác với 1782 phải là chẵn, có một ước nguyên tố lũy thừa 1 và có thể có một ước nguyên tố với một số mũ đồng dư với 1 mod 4, với các ước nguyên tố lẻ khác có các lũy thừa chẵn. Ít nhất một ước nguyên tố phải xuất hiện với một số mũ 4 hoặc cao hơn. Cuối cùng, mọi phản ví dụ, nếu có, phải lớn hơn 1011 . 1.2 Một số ký hiệu và kiến thức chuẩn bị Giả sử n = pα1 1 · · · pαr r > 1 là một số nguyên dương và giả sử k > 1 là một số nguyên nào đó. Trong suốt chương này là sẽ sử dụng các ký hiệu sau: • p1 , p2 , . . . — dãy các số nguyên tố; • σ(n) là hàm tổng các ước; 11 • γ(n) là tích của các ước nguyên tố phân biệt của n; • nếu p là số nguyên tố vp (n) là lũy thừa cao nhất của p mà chia hết n; • ω(n) là số các ước nguyên tố phân biệt của n; • K là tập hợp tất cả những nghiệm của σ(n) = γ(n)2 ; • σk (n) — tổng quát hóa của σ(n), được định nghĩa bởi σk (n) = r (α +1)k Y p i −1 i pki − 1 i=1 ; • ψk (n) — tổng quát hóa của ψ(n), được định nghĩa bởi ψk (n) = nk Y 1 (1 + k ); p p|n • ζ(s) — hàm zeta Riemann, −1 Y 1 ζ(s) = 1− s , p p  • φ(n) — hàm Euler, φ(n) = n Q s = σ + it ∈ C và σ > 1;  1− ; 1 p p|n k • φk (n) — hàm Jordan bậc k, φk (n) = n Q 1− p|n 1 pk  ; • γ — hằng số Euler,   1 1 γ = lim 1 + + . . . + − log n ; n→∞ 2 n • φ∗ (n) — tương tự đơn của φ(n), k Y φ (n) = (pαi i − 1); ∗ i=1 • σ ∗ (n) — tương tự đơn của σ(n), k Y σ (n) = (pαi i + 1). ∗ i=1 12 Định nghĩa 1.2.1. Cho trước một số nguyên dương n. Số nguyên dương d được gọi là một  • ước đơn của n nếu d | n và gcd d, nd = 1; • ước đơn thực sự của n nếu nó là ước đơn và thỏa mãn 1 < d < n; Định nghĩa 1.2.2. Một số nguyên n > 1 được gọi là một số luỹ thừa (powerful integer) nếu nó chia hết cho bình phương của mỗi ước nguyên tố của nó. Một số luỹ thừa còn được gọi là một số bình phương đầy đủ. Định nghĩa 1.2.3. Một hàm f được gọi là hàm nhân tính nếu thỏa mãn f (1) = 1 và f (mn) = f (m)f (n) trong đó m và n là các số nguyên tố cùng nhau. 1.3 Cấu trúc của nghiệm Trước hết chúng ta hãy mô tả dạng của những phần tử trong K. Bổ đề 1.3.1. Nếu n > 1 thuộc K, thì e n = 2 p1 s Y pai i , i=2 trong đó e > 1 và ai chẵn với mọi i = 3, . . . , s. Hơn nữa, hoặc a2 là chẵn, và trong trường hợp đó p1 ≡ 3 (mod 8), hoặc a2 ≡ 1 (mod 4) và p1 ≡ p2 ≡ 1 (mod 4). Chứng minh. Trước hết ta chú ý rằng n phải chẵn: thật vậy nếu n > 1 thỏa mãn σ(n) = γ(n)2 và n là lẻ, thì σ(n) phải lẻ, do đó số mũ của mỗi ước nguyên tố của n phải chẵn, nên n là một bình phương đầy đủ. Nhưng n < σ(n) = γ(n)2 6 n, điều này là mâu thuẫn. 13 Thứ hai, do n là chẵn, ta có 22 k γ(n)2 . Ta viết e n=2 s Y pai i i=1 với các số nguyên tố lẻ phân biệt p1 , . . . , ps và các số mũ nguyên dương a1 , . . . , as , trong đó các số nguyên tố được sắp xếp sao cho các số mũ lẻ xuất hiện đầu tiên và các số mũ chẵn xuất hiện cuối cùng. Sử dụng sự kiện s Q σ(2e ) = 2e+1 − 1 là lẻ, ta nhận được 22 k σ(pai i ). Do đó có nhiều nhất hai i=1 chỉ số i sao cho σ(pai i ) là chẵn, với tất cả những chỉ số khác là lẻ. Nhưng nếu p là lẻ và σ(pa ) cũng là lẻ, thì a là chẵn. Do đó, hoặc chỉ có a1 là lẻ, hoặc chỉ a1 và a2 là lẻ. Bây giờ chúng ta sẽ chứng minh rằng tồn tại ít nhất một số mũ là 1. Giả sử không phải như vậy, lập luận ở trên chỉ ra rằng a1 > 3 và ai > 2 với i = 2, . . . , s. Do đó, s s s Y Y Y 2 3 2 2 3 2 p2i , σ(pi ) > 3p1 pi = γ(n) = σ(n) > σ(2)σ(p1 ) 4p1 i=2 i=2 i=2 điều này dẫn đến p1 < 4/3, đây là một điều vô lý. Do vậy a1 = 1. Cuối cùng, nếu a2 là chẵn, thì 22 k σ(p1 ) chỉ ra rằng p1 ≡ 3 (mod 8), trong khi nếu a2 là lẻ, thì 2 k σ(p1 ) và 2 k σ(pa22 ), dẫn đến kết luận rằng p1 ≡ p2 ≡ 1 (mod 4) và a2 ≡ 1 (mod 4). 1.4 Nghiệm với ω(n) 6 4 Định lí 1.4.1. Giả sử n ∈ K với ω(n) 6 4. Khi đó n = 1 hoặc n = 1782. Chứng minh. Sử dụng Bổ đề 1.3.1, ta viết n = 2α pm, trong đó α > 0 và m là số nguyên tố cùng nhau với 2p. Đầu tiên chúng ta xét trường hợp p = 3. Nếu bổ sung m = 1, thì ta nhận được σ(n) = 62 , và không có nghiệm. Mặt khác, nếu m > 1, thì σ(m) là một ước của γ(n)2 /4 và do đó phải là lẻ. Điều này có nghĩa rằng mọi ước nguyên tố của m xuất hiện với một số mũ chẵn, q β k m. Khi đó σ(q β ) = q β + · · · + q + 1 14 là nguyên tố cùng nhau với 2q và lớn hơn 32 + 3 + 1 > 9. Do đó, tồn tại một ước nguyên tố của m khác 3 hoặc q, gọi là r, mà chia hết q β + · · · + q + 1, điều này kéo theo rằng nó cũng chia hết m và xuất hiện trong phân tích của m với một số mũ chẵn. Do ω(n) 6 4, ta có m = q β rγ . Bây giờ q β + · · · + q + 1 = 3i rj và rγ + · · · + r + 1 = 3k q ` , trong đó i + k 6 2 và j, ` ∈ {1, 2}. Như vậy, (q β + · · · + q + 1)(rγ + · · · + r + 1) = 3i+k q ` rj . Vế trái của đẳng thức này là lớn hơn hoặc bằng 3q β rγ . Trong trường hợp β > 2, ta có β > 4, do đó q 4 r2 6 q β rα 6 9q 2 r2 , điều này dẫn ra q 6 3, đây là điều mâu thuẫn. Mâu thuẫn tương tự cũng nhận được nếu γ > 2. Như vậy, β = γ = 2. Nếu l = j = 2, thì ta nhận được (q 2 + q + 1)(r2 + r + 1) = 3i+k q 2 r2 , điều này dẫn đến σ(2α ) | 32−i−j . Khả năng duy nhất là α = 1 và i + j = 1, nó chứng minh rằng i = 0 hoặc j = 0. Do bài toán là đối xứng, ta chỉ xử lý trường hợp i = 0. Trong trường hợp đó, ta nhận được q 2 + q + 1 = r2 , tương đương với (2q + 1)2 + 3 = (2r)2 , mà không có nghiệm (q, r). Nếu j = ` = 1, thì ta nhận được q 2 r2 < (q 2 + q + 1)(r2 + r + 1) < 9qr, suy ra qr < 9, điều này sai. Do đó, ta xét trường hợp j = 2 và ` = 1, và ngược lại. Do bài toán là đối xứng theo q và r,ta chỉ quan tâm j = 2 và ` = 1. Trong trường hợp này, ta có q 2 r2 < (q 2 + q + 1)(r2 + r + 1) = 3i+k r2 q, do đó q < 3i+k . Do q > 3, điều này chứng tỏ i = k = 1 và q ∈ {5, 7}. Do vậy, r2 + r + 1 = 75, 147, và không xác lập nghiệm n. 15 Từ đây về sau, ta có thể giả thiết p > 3, do đó p + 1 = 2u m1 , trong đó u ∈ {1, 2} và m1 > 1 là lẻ. Giả sử q là ước nguyên tố lớn nhất của m1 . Rõ ràng, p + 1 > 2q, do đó q < p. Hơn nữa, do ω(n) 6 4 ta có p < 4q 4 < q 6 , sao cho q > p1/6 . Giả sử β là số mà q β k n. Ta có thể chứng minh rằng β 6 77. Thật vậy, giả sử β > 78, đầu tiên ta quan sát rằng p13 < q 78 6 q β < σ(q β ), và viết σ(q β ) = 2v m2 , trong đó v ∈ {0, 1} và m2 nguyên tố cùng nhau với 2q. Nếu m2 chia hết p2 , ta nhận được p13 < σ(q β ) 6 2p2 , đây là điều mâu thuẫn. Do đó, tồn tại một ước nguyên tố r khác của n, và m2 6 p2 r2 . Do vậy, p13 < σ(q β ) < 2p2 r2 < p3 r2 , suy ra r > p5 . Giả sử γ thỏa mãn rγ k n. Khi đó r + 1 6 σ(rγ ) 6 2p2 q 2 < p5 , mà đây là điều mâu thuẫn. Như vậy β 6 77. Giả sử r không xuất hiện trong phân tích của (p + 1)σ(q β ). Khi đó ta cần giải hệ phương trình p + 1 = 2u q w và q β + · · · + q + 1 = 2v pz , trong đó β ∈ {1, . . . , 77}, u ∈ {1, 2}, 0 6 v 6 2 − u, {w, z} ⊆ {1, 2}, mà chúng ta có thể giải với kết thức. Điều này cho chúng ta một số khả năng nào đó của các cặp (p, q). Nếu ω(n) = 3, ta có σ(n) = 4p2 q 2 , và ta tìm được 16 n. Nếu ω(n) = 4, thì σ(rγ ) là một ước của 2p2 q 2 và ta tìm các khả năng đối với cặp (r, γ). Sau đó, suy được n từ quan hệ σ(n) = 4p2 q 2 r2 . Bây giờ giả sử r xuất hiện trong phân tích của (p + 1)σ(q β ). Ta viết p + 1 = 2u q w rδ và σ(q β ) = 2v pz rη , (1.2) trong đó u ∈ {1, 2}, w ∈ {1, 2}, 0 6 v 6 2 − u, z ∈ {0, 1, 2}, δ + η ∈ {1, 2}. Nếu z = 0, thì do q > p1/6 , ta có q < σ(q β ) 6 2r2 < r3 , do đó r > q 1/3 > p1/18 . Bây giờ γ 6 89, nếu không thì p5 < r90 6 rγ < σ(rγ ) < 2p2 q 2 < p5 , mà điều này sai. Giả sử z > 0. Khi đó q w rδ < p < 4q w rδ (1.3) từ quan hệ đầu tiên của (1.2), trong khi qβ 2q β z < p < 2rη rη (1.4) từ quan hệ thứ hai của (1.2). Nếu z = 1, ta nhận được từ (1.3) và (1.4) r δ+η < 2q β−w và r δ+η q β−w > . 8 Từ bất đẳng thức bên trái ở trên và δ + η > 1, ta có β − w > 1, và khi đó từ bất đẳng thức bên phải, ta có 9r2 > 8rδ+η > q β−w > q, và do đó r2 > 3r > q 1/2 , do vậy r > q 1/4 > p1/24 . Từ đó, ta có γ 6 119, nếu không thì γ > 120 sẽ cho p5 < r120 6 rγ < σ(rγ ) 6 2p2 q 2 < p5 , mâu thuẫn. Cuối cùng, nếu z = 2, ta nhận được từ (1.4) khẳng định sau √ β/2 q β/2 2q √ q β/2−w √ . 4 2 Từ bất đẳng thức bên trái trên đây và bởi vì δ + η/2 > 1/2, ta có β/2 > w, suy ra β/2 − w > 1/2. Do đó, √ √ 4 2r2 > 4 2rδ+η/2 > q β/2−w > q 1/2 và vì thế √ r8 > 32r4 > (4 2rδ+η/2 )2 > q > p1/6 , điều này chứng tỏ rằng r > p1/48 . Suy ra γ 6 239, nếu γ > 240, thì p5 < r240 6 rγ < σ(rγ ) < 2p2 q 2 < p5 , mâu thuẫn. Do đó ta cần giải hệ p + 1 = 2u q w rδ ; σ(q β ) = 2v pz rη ; σ(rγ ) = 2λ ps q t , với 1 6 β 6 77, 1 6 γ 6 239, u ∈ {1, 2}, u + v + λ 6 2, 1 6 w 6 2, w + t 6 2, δ + η ∈ {1, 2}, z ∈ {0, 1, 2} và s ∈ {0, 1, 2}. Hệ này có thể giải được bằng kết thức, nó cho chúng ta những khả năng có thể đối với bộ số (p, q, r). Do σ(n) = 4p2 q 2 r2 , ta rút ra n bởi việc giải phương trình với α, cho trước p, q và r. Không phát hiện một giá trị nguyên của α có nghĩa là việc tìm nghiệm thất bại. Một chương trình máy tính đã thực hiện các bước này và khẳng định kết luận của Định lí 1.4.1. 1.5 Trường hợp n không có ước là luỹ thừa bậc 4 Định lí 1.5.1. Nếu n > 1 thuộc K, thì n phải có ước là lũy thừa bậc bốn. 18 Chứng minh. Ta giả sử kết quả là sai, nghĩa là, tồn tại một số n ∈ K nào đó không có ước là lũy thừa bậc bốn. Bởi Bổ đề 1.3.1 ta có thể viết e n=2 p1 pa22 k Y qi2 , i=1 trong đó a2 ∈ {0, 1}. Giả sử Q = {q1 , . . . , qk }. Ý tưởng chứng minh là khai thác sự kiện tồn tại nhiều nhất hai phần tử q ∈ Q sao cho q ≡ 1 (mod 3). Q Nếu có ba hoặc nhiều hơn các phần tử như vậy, thì 33 sẽ chia hết σ(q 2 ) q∈Q 2 và do đó nó là một ước của γ(n) , mâu thuẫn. Ta bắt đầu bằng việc chỉ ra rằng k 6 8. Để làm điều này, giả sử       Y q = 1 . R = r ∈ Q : gcd σ(r2 ),   q∈Q Khi đó Q σ(r2 ) chia hết p21 (nếu a2 = 0) và p21 p22 nếu a2 > 0. Từ đây suy ra r∈R 2 σ(r ) hoặc là bội của p1 hoặc của p2 với mỗi r ∈ R. Do tồn tại nhiều nhất hai phần tử r mà σ(r2 ) là một một bội của p1 , và nhiều nhất hai phần tử r mà σ(r2 ) là một bội của p2 , ta nhận được #R 6 4. Khi r ∈ Q \ R, ta có, do σ(r2 ) > 9, σ(r2 ) = r2 + r + 1 là một bội của số nguyên tố qir > 3 nào đó với qir ∈ Q nào đó. Bây giờ, do qir là một ước nguyên tố của r2 + r + 1 lớn hơn 3, nó phải thỏa mãn qir ≡ 1 (mod 3). Do ir có thể nhận các giá trị như thế với nhiều nhất hai số nguyên tố phân biệt r, và tồn tại nhiều nhất hai giá trị phân biệt của chỉ số ir , ta nhận được k − #R 6 4, suy ra k 6 8. Đây chính là yêu cầu đang cần chứng minh. Tiếp theo ta viết phương trình σ(n) = γ(n)2 thành  e+1  k    2  2 − 1 Y qi2 + qi + 1 p1 = 4 qi2 p1 + 1 i=1 2 p2δ 2 σ(pa22 ) ! , (1.5) trong đó δ2 = 0 nếu a2 = 0 và δ2 = 1 nếu a2 > 0. Vế trái của (1.5) không vượt quá  e+1 2   2  − 1  Y q + q + 1 < 0.73(2e+1 − 1). 2 4 q q623 (1.6) 19 Đầu tiên ta giả sử rằng a2 = 0. Khi đó vế phải của (1.5) là p21 9 > = 2.25. p1 + 1 4 (1.7) Nếu e = 1, thì vế trái của bất đẳng thức(1.5) là, do (1.6), nhỏ hơn 0.73(22 − 1) < 2.22, mâu thuẫn với chặn dưới với điều kiện trong (1.7). Do đó, e ∈ {2, 3}, và p21 6 0.73(24 − 1) = 10.95, p1 + 1 vì thế p1 6 11. Do p1 ≡ 3 (mod 8), ta nhận được p1 ∈ {3, 11}. Nếu p1 = 11, thì 3 ∈ Q. Nếu p1 = 3, thì do e ∈ {2, 3}, ta nhận được hoặc là 5 hoặc là 7 sẽ thuộc vào Q. Nếu 3 ∈ Q, thì 13 | 32 + 3 + 1, 61 | 132 + 13 + 1 và 97 | 612 + 61 + 1 thuộc Q và là đồng dư với 1 mod 3, mâu thuẫn. Nếu 5 ∈ Q, thì 31 | 52 + 5 + 1, 331 | 312 + 31 + 1 và 7 | 3312 + 331 + 1 thuộc Q, mâu thuẫn. Nếu 7 ∈ Q, thì 7, 19 | 72 + 7 + 1 và 127 | 192 + 19 + 1 thuộc Q, mâu thuẫn. Tiếp theo giả thiết rằng a2 > 0. Khi đó, bởi Bổ đề 1.3.1, p1 ≡ p2 ≡ 1 (mod 4). Do e ∈ {1, 2, 3}, suy ra một trong ba 3, 5, 7 chia hết n. Nếu 3 | n, thì 3 ∈ Q. Nếu 5 | n, và 5 là one của p1 hoặc p2 , thì 3 | σ(p1 pa22 ) | n, trong khi nếu 5 ∈ Q, thì 31 = 52 + 5 + 1 không đồng dư với 1 mod 4 và chia hết n, kéo theo nó thuộc vào Q, và do đó 3 | 312 + 31 + 1 | n. Cuối cùng, nếu 7 | n, thì 7 không thể là p1 hoặc p2 , có nghĩa là 7 thuộc Q và do đó 3 | 72 + 7 + 1, mà suy ra được 3 | n. Tóm lại, luôn xẩy ra trường hợp khi a2 > 0 thì 3 phải chia hết n. Vậy, 13 = 32 + 3 + 1 chia hết n, do đó hoặc là 13 ∈ Q, hoặc không. Nếu 13 6∈ Q, thì 7 | 13 + 1 thuộc Q, trong trường hợp đó 19 | 72 + 7 + 1 chia hết 20 n và không đồng dư với1 mod 4, suy ra 19 ∈ Q và do đó 127 | 192 + 19 + 1 chia hết n và là không đồng dư với 1 mod 4, do đó 127 ∈ Q. Như vậy, ba số 7, 19, 127 thuộc Q, mà đây lại là điều mâu thuẫn. Nếu 13 ∈ Q, thì 61 | 132 + 13 + 1 chia hết n. Nếu 61 là một trong hai số p1 hoặc p2 , thì 31 | σ(p1 pa22 ) và 31 ≡ 3 (mod 4), nên 31 ∈ Q. Tiếp theo 331 | 312 + 31 + 1 là một ước của n và nó không đồng dư với 1 mod 4, suy ra nó thuộc vào Q và do đó 13, 31, 331 thuộc Q, mâu thuẫn. Cuối cùng, nếu 61 ∈ Q, thì 97 | 612 + 61 + 1 là một ước của n. Nếu 97 ∈ Q ta nhận được mâu thuẫn do 13 và 61 thuộc Q, trong khi nếu 97 là một trong hai số p1 hoặc p2 , thì 7 | σ(p1 pa22 ) là một ước của n, và do đó nhất thiết thuộc vào Q, ta lại được một mâu thuẫn. Vậy, phép chứng minh được hoàn thành. 1.6 Đếm các phần tử trong K ∩ [1, x] Giả sử K(x) = K ∩ [1, x]. Định lí 1.6.1. Ước lượng #K(x) 6 x1/4+o(1) đúng khi x → ∞. Chứng minh. Bởi P. Pollack & C. Pomerance [4, Theorem 1.2], ta có #K(x) = x1/3+o(1) khi x → ∞. Bây giờ ta sẽ cải thiện số mũ 1/3 để nó thành 1/4. Ta nhắc lại một số kết quả trong P. Pollack & C. Pomerance [4].
- Xem thêm -

Tài liệu liên quan

Tài liệu xem nhiều nhất